You are on page 1of 1
4. Find all k > 0 for which a strictly decreasing function g : (0, --o0) + (0, +00) exists such that g(x) > kg(x + g{x)) for all positive 2 The answer is f <1 Solution. Every k < 1 satisfies the condition because for any decreasing g, for instance, g(2 gle) > gle + glz)) > kale + 9(2)) holds. Suppose a function g satisfies the condition for some k > 1. Let s = 4, then g(x + g(x)) < sa(x) Define the sequence (rp) by 20 = 2, Zn41 = tn + git). We have gltnsi) < s9(zn), therefore g(zn) < s"g(z). Since = 4, the inequality 2m = tot gto) + 9(21) ++ balen—1) <2 +9(2) + s9(2) +... +8" tg(e) = Set (lt st. be" Nale) < 2+ ale) it follows that g(rp) > gle-+ gle). Thus g(2+ Lg(2}) < syle) for every positive integral n, which is clearly impossible, because g(x + ;L-g(2)) > 0. The contradiction proves that the case k > 1 is impossible, 5. A convex hexagon ABCDEF is given such that AB || DE, BC || EF, CD || FA. The points M, NV, K are common points of the lines BD and AE, AC and DF, CE and BF respectively, Prove that the perpendiculars drawn from M, V, K to hnes AB, CD, EF respectively are concurrent. Solution. We make use of the following Lemma. Let Tbe the common point of extended legs PS and QR of a trapezium PQRS. Then the radical axis of two circles with diameters Pi u QS is the altitude of the triangle ABC’ drawn from Proof. We consider three circles: w; with diameter PA, w» with diameter QS, and w with diameter PQ. ‘The common chord of w and w is the altitude drawn from P to QR, and the common chord of w and w» is the altitude drawn from Q to PR. The common point of these altitudes, that is, the orthocentre of TPQ, has equal degrees with respect tow and w, and therefore belongs to their radical axis. Similarly, their radical axis contains the orthocentre of 1'S. Since the perpendicular drawn from T to PQ contains both the orthocentres, it is the radical axis of wy and w Applying the lemma to the trapezium ABDE, we find that the perpendicular drawn from M to AB is the radical axis of the circles with diameters AD and BE. Considering the trapezia CDFA and EP'BC in the same way, we prove that the perpendicular drawn from N to CD) is the radical axis of the circles with diameters AD and CP’, and the perpendicular drawn from K to EF is the radical axis of the circles with diameters CF and BE. It follows that the three perpendiculars {no two of which are parallel) have a common point: the radical centre of the three circles. 6, We call a positive integer q a convenient denominator for a real number a if Ja — £| < qh; for some integer p. Prove that if two irrational numbers «and have the same set of convenient denominators then either a+ # or a ~ 3 is an integer. Solution. Let a1 < a2 <... be all convenient denominators for the numbers a and 3. For each a; obviously there exists a unique p; auch that Iga ~ pul < gps we eal this ps convenient numerator corresponding 10 4 First we consider the case when 0 ge +1 then pegs = pe +1. Indeed, the first term of the arithmetical progression with difference a < starting from (gx + Ia is less than (py + 1) — 35 it follows that there must be a term differing from py +1 by less than 4. Similarly, pj, 4; = pk + 1, and the statement is proved. Since [gua — pel < Zp and |ge3 — pe| < qq, we get |gx(a — 8)| < § for all k, hence a 5i 8. In the case of arbitrary a and J we consider the numbers qa and 41. Changing signs if necessary, we may assume that 0< {na}, {18} < 4. Since the numbers qa and 41 satisfy the original condition, {q,a} and {41/3} also satisfy it, therefore {ma} = {919}. This means that ga ~ 419 = r is an integer, i.e. the difference a — 3 = = is rational. Suppose & is not an integer. Then } < {#} < 3 for some k. We shall use the fact that for each wand v,0

You might also like